Difference between revisions of "2017 AMC 8 Problems/Problem 4"
(→Solution) |
Icematrix2 (talk | contribs) |
||
Line 6: | Line 6: | ||
==Solution== | ==Solution== | ||
We can approximate <math>7,928,564</math> to <math>8,000,000,</math> and <math>0.000315</math> to <math>0.0003.</math> Multiplying the two yields <math>2400.</math> Thus shows our answer is <math>\boxed{\textbf{(D)}\ 2400}.</math> | We can approximate <math>7,928,564</math> to <math>8,000,000,</math> and <math>0.000315</math> to <math>0.0003.</math> Multiplying the two yields <math>2400.</math> Thus shows our answer is <math>\boxed{\textbf{(D)}\ 2400}.</math> | ||
+ | |||
+ | ==Video Solution== | ||
+ | https://youtu.be/cY4NYSAD0vQ | ||
+ | |||
+ | ~[[User:icematrix2|icematrix2]] | ||
==See Also== | ==See Also== |
Revision as of 00:46, 3 October 2020
Contents
Problem 4
When is multiplied by the product is closest to which of the following?
Solution
We can approximate to and to Multiplying the two yields Thus shows our answer is
Video Solution
See Also
2017 AMC 8 (Problems • Answer Key • Resources) | ||
Preceded by Problem 3 |
Followed by Problem 5 | |
1 • 2 • 3 • 4 • 5 • 6 • 7 • 8 • 9 • 10 • 11 • 12 • 13 • 14 • 15 • 16 • 17 • 18 • 19 • 20 • 21 • 22 • 23 • 24 • 25 | ||
All AJHSME/AMC 8 Problems and Solutions |
The problems on this page are copyrighted by the Mathematical Association of America's American Mathematics Competitions.